Akademisyenler öncülüğünde matematik/fizik/bilgisayar bilimleri soru cevap platformu
0 beğenilme 0 beğenilmeme
3.8k kez görüntülendi

$\sum\limits_{n=1}^{\infty }\dfrac {1} {n}\sin \left( \dfrac {\pi } {n}\right) $ serisi yakınsak mıdır ıraksak mıdır ?

Lisans Matematik kategorisinde (64 puan) tarafından 
tarafından düzenlendi | 3.8k kez görüntülendi

limitini 0 buluyorum bunun yanlış mıyım acaba ? cevap ıraksakmış

integral testi is gorebilir.

Nasil gorebilir? Yani ne ile integral alacagiz?

haklısınız düzenli azalan olmadıgından ıntegral olmaz, kök testi oran testı de olmaz ancak sizin yaptıgınız gıbı karşılaştırma testıyle yapabılırız, şuanda bılgılerım tartışacak kadar zayıf ancak bir light eyvallah çakıp kaçabilirim.

hayır itraz ediyorum integral testini bal gibi uygulayabiliriz sanırım;

image

Yok, ben testi uygulayamayiz demedim zaten. Uygularsak, nasil uygulariz anlamindaydi...

uyguladım ama bakalım ...

3 Cevaplar

0 beğenilme 0 beğenilmeme
En İyi Cevap

Limitinin sifir olmasi yakinsak oldugunu vermez. Sadece iraksaklik testi ile iraksak diyemeyecegimizi soyler.

Fakat bu seri yakinsak.

Ispat:  Seri pozitif ve $x>0$ icin $\sin x<x$ oldugundan $$0 \le \frac1n\sin(\pi/n) \le \frac1n\frac{\pi}n=\pi\frac1{n^2}$$ olur. Bu da Karsilastirma testi ile serinin yakinsak oldugunu verir.

(25.3k puan) tarafından 
tarafından seçilmiş

açıklama için teşekkürler hocam.

Benzeri olarak da bu videoya bakabilirsin.

Ayrica wolfram da yakinsak diyor. Ara ara kontrol edebilirsin bu tarz sitelerden.

$\sum \dfrac {1} {n}\leq \sum \sin \left( \dfrac {\pi } {n}\right) $  bu eşitsizlik doğru olmuyor o zaman ama yer değişmiş hali doğru değil mi ? sorduklarım bana bu eşitsizliği söyledi.

1) $x>0$ icin $\sin x<x$ oldugundan $sin(\pi/n)< \pi/n$ olur.

2) Bunu diyen kisinin sebebi nedir?

3) Diyelim ki dogru, bu toplamin buyuk olmasi bize neyi veriyor?

değer verdiğimde eşitsizliğe uymadı. sebebi de şu: $1/n$ serisi ıraksak oluğu için diğeri de ıraksak olur.

Digeri bizim serimiz degil. Yazarken mi hatali yazdin acaba. $\sum1/n$ ve $\sum \sin(\pi/n)$ olarak. 

Eger $\sum1/n$ ve $\sum \frac1n\sin(\pi/n)$ olarak ise zaten yanlis olur, sag tarafta $\sin$'li ifadeler $n=2$'den sonra $1$'den kucuk.

soru aynen yazdığım gibi. onlar ikisini ayırıp yapmışlar sanırım. birde harmonik seri olduğu için ıraksak diyen olmuştu ?

Bence bu seri, sıkıştırma teoremi yardımıyla ıraksak olduğunu yazabilirim diye düşünüyorum

0 beğenilme 0 beğenilmeme

image
fonksiyon grafiği budur, düzenli azaldıgından sürekli oldugundan ve pozitiv oldugundan integral testine tabi tutabiliriz,

$\displaystyle\int \dfrac{sin\left(\frac{\pi}{x}\right)}{x}dx$

$u=\dfrac{\pi}{x}$

$-dx=\dfrac{\pi}{x^2}du=\dfrac{u}{x}du$  olur yerlerine koyarsak  

$-\displaystyle\int \dfrac{sin\left(\frac{\pi}{x}\right)}{x}dx=-\displaystyle\int \dfrac{sinu}{u}du$ olur buradan sonrası $-Si(x)$  fonksiyonunun işi,


peki,

$\boxed{\boxed{-\displaystyle\int \dfrac{sin\left(\frac{\pi}{x}\right)}{x}dx=-\displaystyle\int \dfrac{sinu}{u}du}}$

integrali yakınsak mıdır ıraksak mıdır? tabiki de yakınsaktır,

Çünki

$\displaystyle\int_0^\infty \dfrac{sinu}{u}du=\dfrac{\pi}{2}$  olduğunu biliyoruz,


Sınırları "$u$" ya göre revize edip tekrar yazalım,

$\lim\limits_{x\to \infty} \dfrac{\pi}{x}=u=0$  olur , dolayısıyla üst sınır $0$


$\lim\limits_{x\to 0} \dfrac{\pi}{x}=u=\infty$    dolayısıyla alt sınır $\infty$ peki sınırları değiştirirsek integral işaret değiştiriyordu o zaman ,

$\boxed{\boxed{\displaystyle\int_0^\infty \dfrac{\sin\left(\frac{\pi}{x}\right)}{x}dx=-\displaystyle\int_\infty^0 \dfrac{\sin u}{u}du=\displaystyle\int^\infty_0 \dfrac{\sin u}{u}du}}$    olur

Ve 

$\displaystyle\int_0^\infty \dfrac{sinu}{u}du=\dfrac{\pi}{2}$  bu referanstan dolayı , serimiz ıraksakmıştır.

(7.8k puan) tarafından 
tarafından düzenlendi
Son biliyoruz denilen integralin $\pi/2$ oldugunu nereden biliyoruz.

internette yazdım çıktı :) hem sitede de yapmışlardı, @Enis yapmıştı sanırım

bir de soruda 1den başlanmış orayı nasıl törpüleyecegiz?

O cevaplar kompleks analiz ile. Fakat yakinsakligini kismi integral ile gosterebiliriz. $\cos(x)/x^2$ integrali yakinsar falan.

Hic sinirlari yazmamissin. Pek hangi sinirlara gidecegimizi anlamadim. Fakat daha genis aralikta yakinsak olan zaten yakinsak olur, torpulersin yani.

aynen u için değişen sınırları yazmadım ,bilerek yazmadım kim uyanıcak diye :) tabiki de siz farkettiniz :) şüphem yoktu .

düzeltildi.         

0 beğenilme 0 beğenilmeme
$\sum ^{\infty }_{n=1}\dfrac{\pi }{n^{2}}$ yakınsaktır. Limit karşılaştırma testinin sonucu 1 geleceğinden bu seriye yakınsak diyebiliriz.
(234 puan) tarafından 
20,200 soru
21,728 cevap
73,275 yorum
1,887,871 kullanıcı